The ratio of boys and girls in the class is 4:3. How many boys and girls are in the class if there are 35 students?​

Answers

Answer 1
Answer: Boy = 20 and Girl = 15

Explanation:

Boy = 35 x 4/7 = 20

Girl = 35 x 3/7 = 15
Answer 2

Answer:

20boys and 15girls

Step-by-step explanation:

Let no of boys be 4x

no of girls be 3x

4x+3x=35

7x=35

x=35/7

x=5

no of boys=4×5=20

no of girls=3×5=15

20+15=35 students


Related Questions

Type the correct answer in the box. Use numerals instead of words. If necessary, use / for the fraction bar. Stacy goes to the county fair with her friends. The total cost of ride tickets is given by the equation c = 3.5t, where c is the total cost of tickets and t is the number of tickets. If Stacy bought 15 tickets, she would spend $

Answers

Answer:

$52.2

Step-by-step explanation:

Given her total cost of ride tickets modeled by the equation c = 3.5t where c is the total cost of tickets and t is the number of tickets, If Stacy bought 15 tickets, to know the amount she would spend on 15 tickets, we will substitute t = 15 into the modeled equation as shown;

[tex]c = 3.5t\\when t = 15\\\\c = 3.5(15)\\\\c = \frac{35}{10} * 15\\ \\c = \frac{5*7}{5*2} * 15\\\\[/tex]

[tex]c = \frac{7}{2} * 15\\ \\c = \frac{105}{2}\\ \\c = \ 52.2[/tex]

Hence Stacy would spend $52.2 on 15 tickets

Answer:

I hope this helps!

Step-by-step explanation:

Evaluate a + b for a= 34 and b= -6

Answers

Answer:

Hey there!

a+b

34+(-6)

34-6

28

Let me know if this helps :)

Find the measure of each side indicated. Round to the nearest tenth.

A) 19.8
C) 24.9
B) 27.2
D) 25.3

Answers

Answer:

D. 25.3

Step-by-step explanation:

tan∅ = opposite over adjacent

Step 1: Write equation

tan66.5° = x/11

Step 2: Multiply both sides by 11

11tan66.5° = x

Step 3: Evaluate

x = 25.2983

x ≈ 25.3

Answer:

[tex]\huge\boxed{x = 25.3}[/tex]

Step-by-step explanation:

Tan θ = opposite / adjacent

Where θ = 66.5 , opposite = x and adjacent = 11

Tan 66.5 = x / 11

2.3 * 11 = x

25.3 = x

OR

x = 25.3

Greyson completes a dive from a
cliff 75-feet above a river. It takes
him only 1.5 seconds to hit the
water and then another 0.5
second to descend 10 feet into the river

what’s the x axis and y axis?

Answers

Answer: y: height, x: time.

Step-by-step explanation:

The data we have is:

The initial position of Greyson is 75ft above the river.

He needs 1.5 seconds to hit the water, and other 0.5s tho reach the bottom of the river.

Then we have a relationship of height vs time.

The y axis will represent the heigth of Greyson, and the x-axis will represent the time, such that at the time x = 0 seconds, we have y = 75ft

The width of a rectangle measures (6.8d-4.2)(6.8d−4.2) centimeters, and its length measures (9.2d+8.7)(9.2d+8.7) centimeters. Which expression represents the perimeter, in centimeters, of the rectangle?

Answers

Answer:

The perimeter of the rectangle is represented by [tex]p = 32\cdot d + 9[/tex], measured in centimeters.

Step-by-step explanation:

The perimeter ([tex]p[/tex]) of a rectangle, measured in centimeters, is represented by this formula:

[tex]p = 2\cdot (w+l)[/tex]

Where [tex]w[/tex] and [tex]l[/tex] are width and length, measured in centimeters.

If [tex]w = 6.8\cdot d-4.2[/tex] and [tex]l = 9.2\cdot d+8.7[/tex], the expression that represents the perimeter is:

[tex]p = 2\cdot (16\cdot d +4.5)[/tex]

[tex]p = 32\cdot d + 9[/tex]

The perimeter of the rectangle is represented by [tex]p = 32\cdot d + 9[/tex], measured in centimeters.

What is the rate of change and initial value for the linear relation that includes the points shown in the table?
ху
1 20
3 10
5 0
7 -10
Initial value: 20, rate of change: 10
Initial value: 30, rate of change: 10
Initial value: 25, rate of change: -5
Initial value: 20, rate of change: -10

Answers

Answer:

Initial Value: 25, Rate of change -5.

First. Lets find the rate of change.

y2-y1/x2-x1 = m

We have A(1,20) B(3,10)

10-20/3-1=-5

m=-5(Rate of change)

Now let's find the initial value using slope-point form.

y-y₁=m(x-x₁)

y-20=-5(x-1)

=-5x+5+20

=-5x+25

The initial value is the value of y when the value of x is equal to 0. (Also the Y-Intercept)

Initial Value = -5(0)+25

=25

1: The best statement for reason 6 of this proof is -∠A ≅ ∠C
-∠B ≅ ∠D
-∠B and ∠D are supplements
-∠B ≅ ∠B

2.The best reason for statements 3.5. and 7 in this proof is
- Alternate interior angles are congruent.
-Corresponding angles are congruent.
-Alternate exterior angles are congruent.
-Interior angles on the same sides of a transversal are supplements.

3. The best statement for reason 8 of this proof is
-∠B ≅ ∠B -∠A and ∠C are supplements.
-∠B ≅ ∠D
-∠A ≅ ∠C

Answers

Answer:

1) -∠B ≅ ∠D

2) -Interior angles on the same side of a transversal are supplementary

3) -∠A ≅ ∠C

Step-by-step explanation:

1) Given that ∠A and ∠B are supplements and ∠A and ∠D are supplements, we have; ∠B ≅ ∠D

2)  Given that ABCD is a parallelogram, therefore ∠A and ∠B,  ∠A and ∠D and ∠B and ∠C  are interior angles on the same side of a transversal and are therefore supplementary

3) Given that ∠A and ∠B and ∠B and ∠C are supplementary, therefore, ∠A ≅ ∠C.


31. Each day, Talisa exercises by first
stretching and then swimming
some laps, as shown in the table.
Make a scatter plot of the total
time she exercises as a function
of the number of laps she swims.
Draw a trend line.

Answers

Answer:

Step-by-step explanation:

Given the following :

Laps - - - - - - - - 5 - - - 6 - - - 7 - - - 8 - - - 9

Total time - - - 25 - - 28 - - 29 - - 30 - - 32

Using online graphing tool:

The y - axis named dependent variable represents the total time taken.

The x-axis, represents the number of laps.

The equation of the trend line attached to the plot is in the form :

y = mx + c

y = 1.6x + 17.6

Where y = total time taken

x = number of laps

m = 1.6 = gradient of the line (change in y / change in x)

C = 17.6 = intercept (whee the trndline intersects the y-axis).

Complete the following two-way frequency table.

Answers

Answer:

Step-by-step explanation:

Number of candies with Forest = 12

Candies containing coconut and chocolate both = Number common in coconut and the chocolate = 3

Candies which do not contain coconut but contain the chocolate = 6

Candies which contain the coconut but do not contain the chocolate = 1

Candies which neither contain the chocolate nor coconut = 2

From the given Venn diagram,

                                                Contain coconut         Do not contain coconut

Contain chocolate                              3                                       6

Do not contain chocolate                 1                                        2

how many are 8 raised to 4 ???​

Answers

2 cause you divide it

Can you please Solve for x

Answers

x - 3 = 27

add three to both sides

then x= 30

●✴︎✴︎✴︎✴︎✴︎✴︎✴︎✴︎❀✴︎✴︎✴︎✴︎✴︎✴︎✴︎✴︎✴︎●

         Hi my lil bunny!

❧⎯⎯⎯⎯⎯⎯⎯⎯⎯⎯⎯⎯⎯⎯⎯⎯⎯⎯⎯⎯⎯⎯⎯⎯⎯⎯⎯⎯⎯⎯⎯⎯⎯⎯⎯⎯⎯⎯☙

Let's solve your equation step-by-step.

[tex]x-3=27[/tex]

Step 1: Add 3 to both sides.

[tex]x -3 + 3 = 27 +3[/tex]

[tex]x = 30[/tex]

So the answer is : [tex]x = 30[/tex]

❧⎯⎯⎯⎯⎯⎯⎯⎯⎯⎯⎯⎯⎯⎯⎯⎯⎯⎯⎯⎯⎯⎯⎯⎯⎯⎯⎯⎯⎯⎯⎯⎯⎯⎯⎯⎯⎯⎯☙

●✴︎✴︎✴︎✴︎✴︎✴︎✴︎✴︎❀✴︎✴︎✴︎✴︎✴︎✴︎✴︎✴︎✴︎●

Hope this helped you.

Could you maybe give brainliest..?

❀*May*❀

Find the measure of b.

Answers

Answer:

b = 80°

Step-by-step explanation:

The inscribed angle measuring 100°, is supplementary to the angle opposite it in the inscribed quadrilateral.

Thus, the angle is = 80°

Therefore, b + 80° = 180° (angle on a straight line = 180°).

Thus, b = 180° - 80° = 100°.

The measure of b is 80°.

Can any kind soul help me please



Use the cosine rule​

Answers

Answer:

M = 45°

Step-by-step explanation:

a²=b²+c²-2bc Cos A

m² = n² + p² - 2np cos M

(5.1)² = (7.2)² + (5.3)² - 2(7.2)(5.3) cos M

26.01 = 51.84 + 28.09 - 76.32 cos M

-53.92 = -76.32 cos M

0.7065 = cos M

M = [tex]cos^{-1 }[/tex](0.7065)

M = 45°

A laundry basket contains 18 blue socks and 24 black socks. What is the probability of randomly picking 2 black socks, with replacement, from the basket?

Answers

Answer:

144/441

Step-by-step explanation:

There are 18+24=42 total socks

There are 24 black socks

So the probability is (24/42)*(24/42)=12/21 * 12/21 = 144/441

Answer:

189

Step-by-step explanation:

A truck costs $8,000 with a residual value of $1,000. It has an estimated useful life of 7 years. If the truck was bought on July 9 what would be the book value at the end of year 1?

Answers

Answer:

$7520.55

Step-by-step explanation:

Cost of truck = $8000

Residual value = $1000

Estimated useful life = 7 years

Depreciation = (cost of asset - salvage value) / useful life

Depreciation = (8000 - 1000) / 7

Depreciation = 7000 / 7

Depreciation = $1000

Truck was purchased on July 9, Therefore, Depreciation by the end of year one will be;

Number of days between July 9 and year end = 175 days

Daily Depreciation = $1000 / 365 = $2.739

Total Depreciation by year end = (daily Depreciation * 175 days) = $479.45.

Book value at the end of year 1 = (8000 - 479.45)

= $7520.55

Answer:

The answer is "$7,500".

Step-by-step explanation:

Formula:

In the month of July-December the depreciation:

[tex]\frac{\text{Cost-Residual}}{\text{Useful life}}\times \frac{6}{12} \\[/tex]

Cost-Residual= costs -residual value

Given:

Cost-Residual = $8, 000 - $ 1,00

                        = $7,000

Useful life= 7 years

Put the value in the above-given formula:

[tex]=\frac{7 000}{7}\times \frac{6}{12}\\\\= 1 000\times \frac{1}{2}\\\\= 5 00\\[/tex]

Therefore, the book value on point at the end of one year is:

= $8,000 -$ 500

= $7,500

The cost of milk is modeled by a linear equation where four quarts (one gallon) costs $3.09 while two quarts
(half-gallon) costs $1.65. Write the linear equation that expresses the price in terms of quarts. How much would
an eight-quart container of milk cost?

Answers

Answer:

linear equation to express the price is:

y=0.72x+0.21

An eight quarts will cost :  $5.97

Step-by-step explanation:

linear equation represent y=mx+b

let x=quarts ( x=4, x=2)

y= price (3.09 and y=1.65 )

two points (4,3.09) and (2,1.65)

need to find the slope m:

y2-y1/x2-x1

(1.65-3.09)/(2-4) ⇒ m=0.72

y=0.72x+b  find b at point (2,1.65)

1.65=0.72(2) +b  ⇒ b=0.21

y=0.72x +0.21

check : point (4,3.09)

y=0.72(4) +0.21

y=3.09  ( correct)

An eight quarts will cost :

y=0.72(8)+0.21

y=5.97 dollars

Fill in the blank with a number to make the expression a perfect square.
u^2+8u+?

Answers

Answer:

16

Step-by-step explanation:

Hello, do you remember that result?

For any a and b real numbers,

[tex](a+b)^2=a+2\cdot a \cdot b+b^2[/tex]

In this example, we have.

[tex]u^2+8u=u^2+2\cdot 4 \cdot u\\\\\text{ This is the beginning of ... } u^2+8u+4^2=u^2+8u+16\\\\\text{ So, we need to add 16 to make a perfect square}\\\\u^2+8u+\boxed{16}=u^2+2\cdot 4\cdot u +4^2=(u+4)^2[/tex]

Hope this helps.

Do not hesitate if you need further explanation.

Thank you

At the Olympic games, many events have several rounds of competition. One of these events is the men's 100 100100-meter backstroke. The upper dot plot shows the times (in seconds) of the top 8 88 finishers in the final round of the 2012 20122012 Olympics. The lower dot plot shows the times of the same 8 88 swimmers, but in the semifinal round. Which pieces of information can be gathered from these dot plots? (Remember that lower swim times are faster.) Choose all answers that apply: Choose all answers that apply:

Answers

Answer:

The center of the semifinal round distribution is greater than the center of final round distribution.

The variability in the semifinal round distribution is less than variability in the final round distribution.

Step-by-step explanation:

The mean value of each distribution set is not calculates as the center of semifinal round distribution is greater than the final round distribution. MAD Mean Absolute Deviation is calculated from the dotted graph plot, the distribution of semifinal round is less spread out than the final round distribution.

Answer:

correct answer is None of the above i understood nothing the other person was trying to say...

Step-by-step explanation:

mark me brainliest please...

1)Sheyna drive to the lake and back. It took two hours less time to get there than it did to get back. The average speed on the trip there was 60 mph. The average speed on the way back was 36 mph. How many hours did the trip there take?

Answers

Answer:

8 hours

Step-by-step explanation:

Given:

Sheyna drives to the lake with average speed of 60 mph and

[tex]v_1 = 60\ mph[/tex]

Sheyna drives back from the lake with average speed of 36 mph

[tex]v_2 = 36\ mph[/tex]

It took 2 hours less time to get there than it did to get back.

Let [tex]t_1[/tex] be the time taken to drive to lake.

Let [tex]t_2[/tex] be the time taken to drive back from lake.

[tex]t_2-t_1 = 2[/tex] hrs ..... (1)

To find:

Total time taken = ?

[tex]t_1+t_2 = ?[/tex]

Solution:

Let D be the distance to lake.

Formula for time is given as:

[tex]Time =\dfrac{Distance}{Speed }[/tex]

[tex]t_1 = \dfrac{D}{60}\ hrs[/tex]

[tex]t_2 = \dfrac{D}{36}\ hrs[/tex]

Putting in equation (1):

[tex]\dfrac{D}{36}-\dfrac{D}{60} = 2\\\Rightarrow \dfrac{5D-3D}{180} = 2\\\Rightarrow \dfrac{2D}{180} = 2\\\Rightarrow D = 180\ miles[/tex]

So,

[tex]t_1 = \dfrac{180}{60}\ hrs = 3 \ hrs[/tex]

[tex]t_2 = \dfrac{180}{36}\ hrs = 5\ hrs[/tex]

So, the answer is:

[tex]t_1+t_2 = \bold{8\ hrs}[/tex]

8 more than a number

Answers

Answer:

[tex]\boxed{ 8 + x}[/tex]

Step-by-step explanation:

Hey there!

In most cases "the number" would be x.

So if the statement says 8 "more than a number",

It is saying 8 plus x or 8 + x.

Hope this helps :)

Answer:

x + 8 is the meaning.

Step-by-step explanation:

“more” means addition. Take the number as “x”, so it will be x + 8.

That's the answer.

I NEED YOUR HELP PLS

Answers

Answer:

For question 1 you can try dividing each of the value

For instance, you can divide 9 by 25 and see if you get a nice number

e.g. 1/8=0.125, numbers like these

For the second question, you can find the fraction by dividing 1000 starting with the decimal points

e.g 0.650, you would be plotting 650/1000 and you would simplify the fraction to the lowest value any value above the decimal point you can multiply by the denominator and add the nominator value to get your final answer.

Step-by-step explanation:

Answer:

Write the denominator in its prime factors. If the prime factorization of the denominator of a fraction has only factors of 2 and factors of 5, the decimal expression terminates.  If there is any prime factor in the denominator other than 2 or 5, then the decimal expression repeats.

example: 9/25

25 = 5*5, so it will be terminating

example: 7/12

12 = 3*2*2, which contains a 3, so it will be repeating.

Z= -3 - 8i Find the angle θtheta (in degrees) that z makes in the complex plane. Round your answer, if necessary, to the nearest tenth. Express θtheta between -180 180 degrees.

Answers

Answer:  -110.6 degrees approximately

The angle is negative to indicate a clockwise rotation.

======================================================

Explanation:

Z = -3 - 8i is in the form z = a+bi with a = -3 and b = -8

In the complex plane the point (a,b) represents the location of z = a+bi

Define three points with the locations

P = (a,b) = (-3,8)

Q = (0,0)

R = (10,0)

The angle PQR is the angle theta we're looking for. This is the angle formed between the positive x axis and the terminal point (a,b)

Use the arctan function to find theta

theta = arctan(b/a)

theta = arctan( (-8)/(-3) )

theta = 69.4439547804166

theta = 69.4 degrees approximately

Note how this theta value is in quadrant Q1, but (a,b) = (-3, -8) is in Q3

So we need to add 180 degrees to adjust this error.

69.4+180 = 249.4

and we're now in the proper quadrant. We would stop here if your teacher did not put the restriction that theta must be between -180 and 180.

However, this restriction is in place so we need to find the difference of 360 and 249.4 to get 360-249.4 = 110.6

-----------

The angle 249.4 degrees is coterminal to -110.6 degrees. They both point in the same direction.

angle 249.4 degrees is found by starting pointing directly east and rotating 249.4 degrees counterclockwise

angle -110.6 degrees is found by starting directly east and rotating 110.6 degrees clockwise.

Check out the diagram below. I used GeoGebra to make the diagram.

Basic math for 20 points + brainliest!

Answers

Answer:

Look at photo

Step-by-step explanation:

HELP ASAP PLEASE!!!!!!!!!!!!!!!!!

Answers

Answer:

A

C

D

Step-by-step explanation:

√54 or√9 *√6 or √27 *√4

are equal to the answer.

You can do that by doing the square of outer number which is 3 which equals to 9 when squared and multiplying that with the number inside the square root.

Can someone please tell me how to solve this problem??!! I literally have to go back in math if I don’t pass this HELP!!

Answers

Answer:

          D.   270° < φ < 360°

Step-by-step explanation:

Imagine coordinate system

I quarter is where x>0 and y>0  {right top} and it is (0°,90°)

II quarter is where x<0 and y>0  {left top} and it is (90°,180°)

III quarter is where x<0 and y<0  {left bottom} and it is (180°,270°)

IV quarter is where x>0 and y<0  {right bottom} and it is (270°,360°)

Now, we have an angle wich vertex is point (0,0) and one of its sides is X-axis and the second lay at one of the quarters.

For the trig functons of an angle created by this second side always are true:

In first quarter all functions are >0

in second one only sine

in third one: tangent and cotangent

and in fourth one: cosine

{You can check this by selecting any point on the second side of angle and put it's coordinates to formulas of these functions:

[tex]\sin \phi=\dfrac y{\sqrt{x^2+y^2}}\,,\quad \cos \phi=\dfrac x{\sqrt{x^2+y^2}}\,,\quad \tan\phi=\dfrac yx\,,\quad \cot\phi=\dfrac xy[/tex]  }

So:

sinφ<0  ⇒ III or IV quarter

tanφ<0  ⇒ I or IV quarter

IV quarter  ⇒  φ ∈ (270°, 360°)

Reduce 5/15 to its lowest terms

Answers

Answer:

The answer is 1/3

Answer:

1/3

Step-by-step explanation:

The factors of 5 are 1,5;

* The factors of 15 are 1,3,5,15.

We can see that the GCD is 5 because it is the largest number by which 5 y 15 can be divided without leaving any residue.

To reduce this fraction, simply divide the numerator and denominator by 5 (the GCF).

So, 5 /15

= 5÷5 /15÷5

= 1 /3

What is 12.5% of 72

Answers

Answer:

[tex]\boxed{9}[/tex]

Step-by-step explanation:

[tex]\sf of \ refers \ to \ multiplication.[/tex]

[tex]12.5\% \times 72[/tex]

[tex]\frac{12.5}{100} \times 72[/tex]

[tex]\sf Multiply.[/tex]

[tex]\frac{900}{100} =9[/tex]

Find the missing probability. P(A)=1120,P(B|A)=1320,P(A∩B)=?

Answers

Answer: 143/400

Explanation:

Assuming you meant to say

P(A) = 11/20

P(B|A) = 13/20

then,

P(A∩B) = P(A)*P(B|A)

P(A∩B) = (11/20)*(13/20)

P(A∩B) = (11*13)/(20*20)

P(A∩B) = 143/400

THE ANSWER IS 143/400 !

I need help on both answers. They’re different from my other problems so I’m kinda confused

Answers

i think it would be b but
i’m not sure about the other one

Find the distance across the lake. Assume the triangles are similar.
80 m
х
у
20 m
60 m

Answers

Answer:

a

Step-by-step explanation:

Answer:

A.   L = 240 m

Step-by-step explanation:

use similar triangle

L / 60 = 80 / 20

L = (80 * 60) / 20

L = 240 m

Other Questions
If youre writing your own piece of literature which strategy would make your work seem more postmodern in character A. Use rhythm and rhyme in very structured, highly predictable way. B. Use elements of popular culture to consider serious topics or themes. C. Rely on traditional forms and their conversations to guide your expressions of ideas D. Focus on the elements of narrative that readers expect to see in all contemporary work. An electron moving at 3.94 103 m/s in a 1.23 T magnetic field experiences a magnetic force of 1.40 10-16 N. What angle does the velocity of the electron make with the magnetic field? There are two answers between 0 and 180. (Enter your answers from smallest to largest.) Identify five HRM criteria or components that can be used to measure organizational effectiveness or ineffectiveness. "Grievance rate" is an example. 5. Choose the two positives of the Columbian exchangeNative Americans spread diseases to European explorersEuropeans introduced horses, livestock, guns and the wheel to native societyEuropeans unknowingly brought many deadly diseases to the New WorldEuropeans brought back native crops (potato, corn, beans, etc) to Europe leading to better nutrition The Fairy Tale Spectacular is coming to town. Admission to the fair costs $32.50 and each ride costs $0.80. You have $50 to spend at the Fairy Tale Spectacular including admission. Write and solve an inequality to determine the maximum number of rides you can enjoy at the Fairy Tale Spectacular? If the range of feasibility indicates that the original amount of a resource, which was 20, can increase by 5, then the amount of the resource can increase to 25.a. Trueb. False If the normality requirement is not satisfied (that is, np(1p) is not at least 10), then a 95% confidence interval about the population proportion will include the population proportion in ________ 95% of the intervals. (This is a reading assessment question. Be certain of your answer because you only get one attempt on this question.) Input resistance of a FET is very high due to A) forward-biased junctions have high impedance B) gate-source junction is reverse-biased C) drain-source junction is reverse-biased D) none of the above Todays business headlines frequently cite pensions being underfunded, thus costing companies more in contributions to their pension fund as well as pensioners risking not receiving what they had planned for retirement. This has been caused by underperformance of the pension fund itself and the over promising of benefits to retirees. Take the following example:_______. Assume $20m was invested today to provide for pension payments for a group of employees. Assume also that the average return on these funds was 8.5% 1. How big will the fund be in 25 years? 2. Suppose at year 12 the fund decreased in value by 30%. What returns would be required for the next 13 years to achieve the 25 year amount? 3. Advisor's counseled the company that a conservative investment return of 6% annually for the next 13 years would be advisable and that the company would have to contribute annually to make up the shortfall. How much would have to be contributed annually beginning year 13 if the fund earned 6% in order to achieve the 25 year goal? Please show the method used to solve this problem. please help solving. What is the pH of a solution made by mixing 15.00 mL of 0.10 M acetic acid with 15.00 mL of 0.10 M KOH? Assume that the volumes of the solutions are additive. K a = 1.8 10-5 for CH3CO2H. If a corporation has a dividend payout ratio of 75%, the undistributed earnings (25%) will:_________. A. increase earnings per share. B. decrease book value. C. increase capital in excess of parD. increase retained earnings A simple structure is characterized by Multiple Choice high specialization and low centralization. low specialization and high centralization. low formality and low creativity. high formality and low centralization. The scientific method requires thatA. The scientist be objective.B. The scientist use precision equipment.C. Only correct theories are tested.D. Only incorrect theories are tested Using the standard reduction potentials Ni2+(aq) + 2 eNi(s) 0.25 volt Fe3+(aq) + eFe2+(aq) +0.77 volt Calculate the value of Ecell for the cell with the following reaction. Ni2+(aq) + 2 Fe2+(aq) Ni(s) + 2 Fe3+(aq) When Production decreases what is a very likely possibility? a hire new workers b expand production c purchase new equipment d downsizing A random number generator chooses 2 numbers between 1 and 5. This table demonstrates all possible outcomes. 1 2 3 4 5 1, 1 1, 2 1, 3 1, 4 1, 5 2 2, 1 2, 2 2, 3 2, 4 2, 5 3 3, 1 3, 2 3, 3 3, 4 3, 5 4 4, 1 4, 2 4, 3 4, 4 4, 5 5 5, 1 5, 2 5, 3 5, 4 5, 5 What is the probability that the computer will pick two 5s? Which is the first step of the creative process?A. PolishingB. BrainstormingC. DraftingD. Revising Which of the following laid the foundation for cartography, the artand science of mapmaking?A. Ancient EgyptiansB. Early EuropeansC. Ancient RomansD. ArabsE. Ancient Greekstosth A firm has the following gross requirements for Item OF. Ordering costs are $60 per order and carrying costs are $0.50 per period.Item F Period LT: 1 1 2 3 4Gross Requirements 60 40 80 60Schedule Receipts Project on Hand 100 Net Requirements Planned Order Receipts Planned Order Releases If EOQ lot sizing is used the minimum order quantity would be:_______a. 85b. 100c. 120d. 150